Regla de Feynman para propagador con derivadas

Suponga que tiene un término de interacción de la forma

L i norte t = h gramo 3 ! ϕ 3 2 ϕ
dónde h y gramo ambos son acoplamientos. Ahora, si dibujo un diagrama de la forma dada en la figura (por favor ignore el λ arena j 's.) Entonces suponga que el propagador es el campo con la derivada en: ¿cuál sería la amplitud en este caso? Sé cómo calcular los factores de vértice, pero este me confunde. ¿Alguna sugerencia?

Si no hubiera un propagador involucrado, simplemente escribiría

S i norte t = i h gramo 3 ! d 4 X ϕ 3 ( X ) 2 ϕ ( X ) = i h gramo 3 ! d 4 X D k ϕ 1234 ~ Exp [ i ( k 1 + + k 4 ) ] ( i k 4 ) 2
que entonces produciría el factor de vértice V de (cuidar de 3 ! permutando los tres ϕ 's)
V = i h gramo k 4 2 .
He tomado todos los momentos como entrantes en el vértice y D k = d 4 k 1 d 4 k 2 d 4 k 3 d 4 k 4 ( 2 π . . .2 π ) 4 la tilde es la transformada de Fourier de cada campo por separado.

Pero esto es para un vértice, ¿qué tal si hay un propagador involucrado?

ingrese la descripción de la imagen aquí

¿Qué has probado? Lo mejor es escribir el vértice en el espacio de Fourier y todo debería seguir.
Estoy un poco confundido por el hecho de que un campo es señalado por la derivada. Debería intentar reescribir la densidad lagrangiana de una manera más simétrica.

Respuestas (2)

El propergador no tiene derivada. El término de interacción es el vértice en el diagrama de Feynman.

A continuación utilizo la convención de notación de Sredniski. La convención de Peskin causaría algún signo menos adicional.

Por ejemplo, ϕ 3 2 ϕ , el vértice es 0 | ϕ 3 2 ϕ | k 1 k 2 k 3 k 4 = 4 2 0 | ϕ ( X 1 ) ϕ ( X 2 ) ϕ ( X 3 ) ϕ ( X 4 ) | k 1 k 2 k 3 k 4 = 4 2 ( mi i k 1 X 1 + i k 2 X 2 + i k 3 X 3 + i k 4 X 4 + pag mi r t tu metro a t i o norte s ) | X i = X = 3 ! ( k i 2 ) mi i ( k 1 + k 2 + k 3 + k 4 ) X

Entonces, el vértice en el espacio de cantidad de movimiento es i gramo / 3 ! 3 ! ( k i 2 ) = i gramo ( k i 2 ) .

El propagador no se verá afectado ya que está completamente determinado por el Lagrangiano de campo libre. Entonces D F = ( 2 + metro 2 ) 1 , en el espacio de momento, 1 k 2 + metro 2

Entonces, si tiene un acoplamiento derivado, tendrá un vértice diferente, pero otras cosas son iguales con lo habitual ϕ 4 acoplamiento.

Por favor, vea mi pregunta actualizada.
Disculpa no entiendo cual es tu pregunta ¿Por qué te confunde la amplitud? ¿Podrías escribir cómo obtienes la amplitud? (Por cierto, quizás cometas un error al derivar el vértice ...)

Aunque esto es 7 años después, noté que hay una brecha en la literatura sobre cómo derivar las reglas de Feynman para este tipo de lagrangianos. Permítanme escribir cómo derivar el vértice para el Lagrangiano que mencionó para ayudar a las personas en el futuro que trabajan en QFT, ya que creo que las otras respuestas en los foros son demasiado confusas para los principiantes y no tan sistemáticas en general. El cálculo de las reglas de Feynman se reduce a la computación de propagadores, a partir de los cuales se puede construir una función generadora e incluir interacciones. Luego siguen las reglas de Feynman tomando un montón de derivadas funcionales.

Así que comencemos con el Lagrangiano:

L i norte t = gramo h 3 ! ϕ 3 2 ϕ .

El enfoque funcional de QFT se puede utilizar para derivar la regla de Feynman del vértice. Permítanme tratar de dar una respuesta/estrategia independiente a tales preguntas. Supongamos que el campo escalar ϕ obedece al lagrangiano estándar de campo libre de Klein-Gordon L 0 = 1 2 ( m ϕ ) 2 1 2 metro 2 ϕ 2 . Entonces el propagador se puede encontrar escribiendo la parte libre de la acción como:

S 0 = d 4 X [ 1 2 ( m ϕ ) 2 1 2 metro 2 ϕ 2 ] = d 4 X [ 1 2 ϕ ( m m metro 2 ) ϕ ] ,

donde usé integración por partes para el primer término. Esto sugiere que el propagador D ( X y ) puede ser encontrado por:

( m m + metro 2 ) D ( X y ) = i d ( X y ) .

El i aquí es sólo una cuestión de convención. En el espacio de Fourier vemos fácilmente que:

D ( k ) = i k 2 metro 2 .

Dejar j ( X ) ser alguna función del espacio-tiempo que llamamos la corriente asociada a ϕ . La función generadora se puede escribir como una integral de trayectoria:

Z [ j ] = D ϕ mi i d 4 X ( L + j ( X ) ϕ ( X ) ) ,

dónde L = L 0 + L i norte t y D ϕ denota la integración sobre todas las configuraciones de campo (para un campo de indicador, por ejemplo, debe tener cuidado al fijar un indicador en esta expresión, pero para el campo escalar no hay problemas). En este lenguaje es sencillo ilustrar que la parte libre de la función generadora se puede escribir de la forma:

Z 0 [ j ] = Z 0 [ 0 ] mi 1 2 d 4 X d 4 y j ( X ) D ( X y ) j ( y ) .

Esto se puede hacer a partir de la definición de Z [ j ] , solo considere el Lagrangiano libre y defina el campo desplazado ϕ ( X ) = ϕ ( X ) i d 4 y j ( y ) D ( X y ) . el subíndice 0 indica que consideramos solo la parte libre.

Para encontrar la expresión del vértice de 4 puntos que escribiste, debes calcular la función de correlación de 4 puntos:

Ω | T { ϕ ( X 1 ) ϕ ( X 2 ) ϕ ( X 3 ) ϕ ( X 4 ) } | Ω = d 4 X ϕ ( X 1 ) ϕ ( X 2 ) ϕ ( X 3 ) ϕ ( X 4 ) mi i d 4 X L d 4 X mi i d 4 X L .

Es sencillo comprobar que en el lenguaje de las derivadas funcionales y la función generadora Z [ j ] uno podría escribir esto como:

Ω | T { ϕ ( X 1 ) ϕ ( X 2 ) ϕ ( X 3 ) ϕ ( X 4 ) } | Ω = 1 Z [ 0 ] ( i d d j ( X 1 ) ) ( i d d j ( X 2 ) ) ( i d d j ( X 3 ) ) ( i d d j ( X 4 ) ) Z [ j ] | j = 0 .

Bien, ahora tenemos todo listo para el cálculo. La función generadora total se puede escribir como:

Z [ j ] = D ϕ mi i d 4 X L i norte t Z 0 [ j ] = D ϕ mi i d 4 X gramo h 3 ! ( i d d j ( X ) ) 3 2 ( i d d j ( X ) ) Z 0 [ j ] .

Si gramo , h son pequeños, entonces podemos ver una expansión perturbativa de este exponencial. Centrándonos en la parte que interactúa en el orden líder, encontramos que:

Z [ j ] i d 4 X gramo h 3 ! ( i d d j ( X ) ) 3 2 ( i d d j ( X ) ) Z 0 [ j ] .

Por lo tanto, la función de correlación se puede escribir como:

Ω | T { ϕ ( X 1 ) ϕ ( X 2 ) ϕ ( X 3 ) ϕ ( X 4 ) } | Ω = ( i d d j ( X 1 ) ) ( i d d j ( X 2 ) ) ( i d d j ( X 3 ) ) ( i d d j ( X 4 ) ) i d 4 X gramo h 3 ! ( i d d j ( X ) ) 3 2 ( i d d j ( X ) ) mi 1 2 d 4 X d 4 y j ( X ) D ( X y ) j ( y ) | j = 0 .

Explotemos la notación en la que dejamos la integral sobre X implícito y por ejemplo D X y = D ( X y ) , j y = j ( y ) , D X y j y d 4 y D ( X y ) j ( y ) para hacer expresiones más concisas. También dejaremos la evaluación en j = 0 implícito. Diagramas de burbujas, es decir, los que incluyen D X X será ignorado en la expansión a continuación. También ignoraremos los términos que darán cero para j = 0 .

Entonces, comencemos a calcular esta función de correlación usando esta notación compacta:

i Ω | T { ϕ ( X 1 ) ϕ ( X 2 ) ϕ ( X 3 ) ϕ ( X 4 ) } | Ω = gramo h 3 ! d d j 1 d d j 2 d d j 3 d d j 4 ( d d j X ) 3 2 ( d d j X ) mi 1 2 j z D z y j y = gramo h 3 ! d d j 1 d d j 2 d d j 3 d d j 4 ( d d j X ) 3 mi 1 2 j z D z y j y 2 ( D X y j y ) = gramo h 3 ! d d j 1 d d j 2 d d j 3 d d j 4 ( d d j X ) 2 j z D X z mi 1 2 j z D z y j y 2 ( D X y j y ) = gramo h 3 ! d d j 1 d d j 2 d d j 3 d d j 4 d d j X ( D X z j z D X v j v mi 1 2 j z D z y j y 2 ( D X y j y ) ) = gramo h 3 ! d d j 1 d d j 2 d d j 3 d d j 4 ( D X z j z D X v j v D X w j w mi 1 2 j z D z y j y 2 ( D X y j y ) ) = gramo h 3 ! d d j 1 d d j 2 d d j 3 [ 3 D X 4 D X v j v D X w j w 2 ( D X y j y ) + D X z j z D X v j v D X w j w 2 ( D X 4 ) ] mi 1 2 j z D z y j y = gramo h 3 ! d d j 1 d d j 2 [ 3 ! D X 4 D X 3 D X w j w 2 ( D X y j y ) + 3 D X 4 D X v j v D X w j w 2 D X 3 + 3 D X 3 D X v j v D X w j w 2 D X 4 ] mi 1 2 j z D z y j y = gramo h 3 ! d d j 1 [ 3 ! D X 4 D X 3 D X 2 2 ( D X y j y ) + 3 ! D X 4 D X 3 D X w j w 2 D X 2 + 3 ! D X 4 D X 2 D X w j w 2 D X 3 + 3 ! D X 3 D X 2 D X w j w 2 D X 4 ] mi 1 2 j z D z y j y = gramo h [ D X 4 D X 3 D X 2 2 D X 1 + D X 4 D X 3 D X 1 2 D X 2 + D X 4 D X 2 D X 1 2 D X 3 + D X 3 D X 2 D X 1 2 D X 4 ] .

En la notación ordinaria encontramos así que:

Ω | T { ϕ ( X 1 ) ϕ ( X 2 ) ϕ ( X 3 ) ϕ ( X 4 ) } | Ω = i gramo h d 4 X [ D ( X X 4 ) D ( X X 3 ) D ( X X 2 ) 2 D ( X X 1 ) + D ( X X 4 ) D ( X X 3 ) D ( X X 1 ) 2 D ( X X 2 ) + D ( X X 4 ) D ( X X 2 ) D ( X X 1 ) 2 D ( X X 3 ) + D ( X X 3 ) D ( X X 2 ) D ( X X 1 ) 2 D ( X X 4 ) ] .

Recuerde que el propagador está dado por:

D ( X y ) = d 4 k ( 2 π ) 4 i k 2 metro 2 mi i k X .

Por lo tanto, ahora es fácil ver que el vértice está dado por:

V ϕ ϕ ϕ ϕ = i gramo h [ pag 1 2 + pag 2 2 + pag 3 2 + pag 4 2 ] ,

dónde pag 1 + pag 2 + pag 3 + pag 4 = 0 (por lo que todos los momentos son entrantes) ya que una función delta del tipo d ( pag 1 + pag 2 + pag 3 + pag 4 ) aparecerá si calcula la función de correlación explícitamente.

Su resultado final se refiere al vértice. ¿Qué pasa con el caso de una interacción de intercambio, como se hizo la pregunta? ¿Se sustituiría una cantidad de movimiento por una cantidad de movimiento interna? ¿O uno simplemente obtendría pag 1 2 + pag 2 2 + pag 3 2 , es decir, los momentos externos solamente?
Uno de los momentos es interno en ese caso y tres externos